Limit Of $(1+ X/n)^n$ When $n$ Tends To Infinity - Math Stack Exchange

    1. Home
    2. Questions
    3. Tags
    4. Users
    5. Unanswered
  1. Teams

    Ask questions, find answers and collaborate at work with Stack Overflow for Teams.

    Try Teams for free Explore Teams
  2. Teams
  3. Ask questions, find answers and collaborate at work with Stack Overflow for Teams. Explore Teams

Teams

Q&A for work

Connect and share knowledge within a single location that is structured and easy to search.

Learn more about Teams Limit of $(1+ x/n)^n$ when $n$ tends to infinity [duplicate] Ask Question Asked 10 years, 4 months ago Modified 6 years, 7 months ago Viewed 380k times 49 $\begingroup$ This question already has answers here: About $\lim \left(1+\frac {x}{n}\right)^n$ (14 answers) Closed 10 years ago.

Does anyone know the exact proof of this limit result?

$$\lim_{n\to\infty} \left(1+\frac{x}{n}\right)^n = e^x$$

Share Cite Follow edited Jul 30, 2014 at 15:32 Darth Geek's user avatar Darth Geek 12.4k1 gold badge34 silver badges52 bronze badges asked Jul 30, 2014 at 15:30 narendra-choudhary's user avatar narendra-choudharynarendra-choudhary 7881 gold badge8 silver badges14 bronze badges $\endgroup$ 6
  • 2 $\begingroup$ Depends on the definition of $e^x$... $\endgroup$ – Thomas Andrews Commented Jul 30, 2014 at 15:32
  • $\begingroup$ have you tried to explicit the power and take the limit? It converge to the taylor series. $\endgroup$ – Lolman Commented Jul 30, 2014 at 15:33
  • $\begingroup$ I tried by taking log of both sides, but I don't know what to do after this step. Thought of using L'Hopital's rule. But that ain't helping. $\endgroup$ – narendra-choudhary Commented Jul 30, 2014 at 15:38
  • $\begingroup$ You can use this technique. $\endgroup$ – Mhenni Benghorbal Commented Jul 30, 2014 at 15:44
  • 5 $\begingroup$ Using L'Hopital on the log you get: $$\lim_{n\to\infty} \frac{\log(1+\frac{x}{n})}{1/n}=\lim_{n\to\infty}\frac{n}{n+x}\frac{-x}{n^2}(- n^2)=x$$ $\endgroup$ – Lolman Commented Jul 30, 2014 at 15:46
| Show 1 more comment

3 Answers 3

Sorted by: Reset to default Highest score (default) Date modified (newest first) Date created (oldest first) 84 $\begingroup$

A short proof:

$\left(1+\frac{x}{n}\right)^n = e^{n\log\left(1+\dfrac{x}{n}\right)}$

Since $\log(1+x) = x + O(x^2)$ when $x \to 0$, we have $n\log(1 + \frac{x}{n}) = x + O(\frac{x^2}{n})$ when $n\to +\infty$

Share Cite Follow edited Jul 30, 2014 at 15:37 Thomas Andrews's user avatar Thomas Andrews 180k18 gold badges219 silver badges413 bronze badges answered Jul 30, 2014 at 15:36 Petite Etincelle's user avatar Petite EtincellePetite Etincelle 14.9k2 gold badges33 silver badges61 bronze badges $\endgroup$ 7
  • 4 $\begingroup$ Or just use the derivate of $\log$ to evaluate $\lim_{y\to 0}\frac{\log(1+xy)}{y}$, then let $y=1/n$ to get the limit of $n\log(1+x/n)$... $\endgroup$ – Thomas Andrews Commented Jul 30, 2014 at 15:39
  • 2 $\begingroup$ Even more elementary would be to use the basic inequality $\frac{x/n}{1+x/n}\le \log(1+x/n)\le x/n$. Then a straightforward application of the squeeze theorem does the trick. $\endgroup$ – Mark Viola Commented Oct 25, 2015 at 16:03
  • 2 $\begingroup$ In this case, why does writing $(1+\frac{x}{n})^n=10^{n\log_{\, 10} \, \, (1+\frac{x}{n})}$ not show that $\lim_{n \to \infty}(1+\frac{x}{n})^n=10^x$? $\endgroup$ – Rasputin Commented Jan 19, 2019 at 21:17
  • 6 $\begingroup$ @Rasputin because the Taylor expansion of $\log_{10} (1+x)$ is different $\endgroup$ – Petite Etincelle Commented Jan 20, 2019 at 13:27
  • 1 $\begingroup$ What is $O(x^2)$? $\endgroup$ – PGupta Commented Sep 19, 2020 at 17:00
| Show 2 more comments 52 $\begingroup$

$$e^{\ln{(1 + \frac{x}{n})^n} }=e^{n \ln(1+\frac{x}{n})}$$

$$\lim_{n \to +\infty} (1 + \frac{x}{n})^n =\lim_{n \to +\infty} e^{n \ln(1+\frac{x}{n})} \\ =e^{\lim_{n \to +\infty} n \ln(1+\frac{x}{n})} =e^{\lim_{n \to +\infty}\frac{ \ln(1+\frac{x}{n})}{\frac{1}{n}}}$$

Apply L'Hopital's Rule:

$$=e^{\lim_{n \to +\infty}\frac{(\frac{-x}{n^2})\frac{1}{1+\frac{x}{n}}}{-\frac{1}{n^2}}} =e^{\lim_{n \to +\infty}\frac{x}{1+\frac{x}{n}}} =e^x$$

Therefore, $$(1+\frac{x}{n})^n \to e^x$$

Share Cite Follow edited May 2, 2018 at 20:27 Community's user avatar CommunityBot 1 answered Jul 30, 2014 at 15:46 evinda's user avatar evindaevinda 7,9596 gold badges42 silver badges88 bronze badges $\endgroup$ 6
  • 2 $\begingroup$ First: Is the first result in the fourth row by L'Hopital's rule? Second, isn't a limit missing in the second result of the fourth row? $exp(x/(1+x/n))$ should be $exp\{lim[x/(1+x/n)]\}$ $\endgroup$ – Carol Eisen Commented Dec 25, 2017 at 17:39
  • 1 $\begingroup$ You are correct! I edited it to account for the missing limit. Also, you are correct in that one of the steps is using L'Hopital's Rule. $\endgroup$ – H. Dewey Commented May 2, 2018 at 19:58
  • $\begingroup$ Should I have permission to apply L' Hospital as it is n tends to infinity not x ,I mean it is kind of discrite case not continuous. $\endgroup$ – Supriyo Banerjee Commented Apr 24, 2019 at 14:53
  • $\begingroup$ I understand everything except the part $\dfrac{-x}{n^{2}}$. Where does this come from? Isn't the derivative of $\ln(1+\frac{x}{n})=\frac{1}{1+\frac{x}{n}}$ only? $\endgroup$ – James Warthington Commented Oct 12, 2019 at 19:21
  • 2 $\begingroup$ you're differentiating with respect to x, not n, my man. The variable in the limit is N not x. $\endgroup$ – astralwolf Commented Dec 23, 2019 at 19:45
| Show 1 more comment 35 $\begingroup$

You can use the binomial series expansion. For example:

$$\left(1+\frac{x}{n}\right)^n =1+ \frac{n}{1!}\left(\frac{x}{n}\right)^1+\frac{n(n-1)}{2!}\left(\frac{x}{n}\right)^2+\frac{n(n-1)(n-2)}{3!}\left(\frac{x}{n}\right)^3+\cdots $$

$$\left(1+\frac{x}{n}\right)^n =1+ \frac{n}{n}x+\frac{n(n-1)}{n^2}\frac{x^2}{2!}+\frac{n(n-1)(n-2)}{n^3}\frac{x^3}{3!} + \cdots$$

As $n \to \infty$ the coefficients in $n$ all tend to $1$. Hence:

$$\lim_{n \to \infty}\left(1+\frac{x}{n}\right)^n = 1+x+\frac{x^2}{2!}+\frac{x^3}{3!}+\cdots $$ You'll recognise this last power series as the Taylor series for $\mathrm{e}^x$.

Share Cite Follow edited Aug 20, 2016 at 19:11 answered Jul 30, 2014 at 15:39 Fly by Night's user avatar Fly by NightFly by Night 32.7k4 gold badges56 silver badges103 bronze badges $\endgroup$ 5
  • 16 $\begingroup$ what if there are infinite terms and then you cannot exchange the sum and limit if n is non integer $\endgroup$ – happymath Commented Mar 3, 2015 at 10:17
  • 3 $\begingroup$ it's not easy to make this argument rigorous. This answer would be better if it pointed out that fact $\endgroup$ – wlad Commented May 30, 2019 at 18:28
  • 4 $\begingroup$ a rigorous version of this argument uses the monotone convergence theorem $\endgroup$ – wlad Commented May 30, 2019 at 18:41
  • $\begingroup$ rigorous version: math.stackexchange.com/a/1898375/445105 $\endgroup$ – Felix Benning Commented Sep 17, 2021 at 10:03
  • $\begingroup$ @happymath $n$ is Never Non Ninteger :-) $\endgroup$ – Oskar Limka Commented Oct 12, 2021 at 17:08
Add a comment |

Not the answer you're looking for? Browse other questions tagged .

  • Featured on Meta
  • We’re (finally!) going to the cloud!
  • More network sites to see advertising test [updated with phase 2]

Linked

55 About $\lim \left(1+\frac {x}{n}\right)^n$ 17 If $z_n \to z$ then $(1+z_n/n)^n \to e^z$ 9 Limit of $(1+1/n)^n$ is not equal to one, but why ? 18 How to calculate the limit $\lim_{n\to\infty}\left(\frac{1}{\sqrt{n^2+n+1}}+\frac{1}{\sqrt{n^2+n+2}}+\cdots+\frac{1}{\sqrt{n^2+n+n}}\right)^n$ 9 Geometric distribution converges to exponential distribution 2 Limit of $\left(1+\frac{1}{2n}\right)^n$ 0 Stuck in the proof of $(1+\frac{x}{n})^n \to e^x$ 2 Definition of e, and proof of existence. 3 Finding the convergence 2 A question regarding the prime-number theorem See more linked questions 0 A limit to infinity 1 The limit of product of two functions, where one tends to infinity and the other to zero 2 Limit of geometric series when common ratio tends to 1 9 limit when zero divided by infinity 0 Calculating limit for sequence that tends to infinity 0 What is the limit of series when n tends to infinity? 2 Find the limit of a multiplying term function when n tends to infinity. 3 L'Hospital when limit tends to infinity 0 Limit of the hypergeometric function with x tends infinity

Hot Network Questions

  • Can you omit から directly connecting plain forms?
  • Prospective employers tell me my field is obsolete. How can I reinvent myself?
  • I have a Greek seasonal residence permit. Can I go back to my country from Poland instead of Greece without any issue?
  • Can not update chrome in ubuntu
  • Criticisms of "reductio ad absurdum" argumentation?
  • The usage of Select for list (deep level)
  • Is it (always) better to build a model prior to viewing the data?
  • PSE Advent Calendar 2024 (Day 1): A Snowy Christmas
  • How to calibrate fancy headers?
  • How do I report to Springer a scientific fraud to a cryptographic paper published in Springer?
  • In AES GCM, would using different nonces that are close reveal data?
  • Calculating the wattage of a speaker protection light bulb
  • To which extent I should let my PI know that I am not feeling very well with my PhD study
  • Is it possible to use NAS hard drives in a desktop?
  • Why should C++ uint8_t data not be printable?
  • Can Trump reverse President Joe Biden's pardon for his son?
  • Use of “12 m.” for noon and “12 p.m.” for midnight
  • What are Christian responses to Carlo Alvaro's argument against Christian theism?
  • Please explain the end of Tosefta Eduyot
  • Why do you want a taut surface on dough?
  • Example of non homogenous manifold with a finitely generated algebra of natural functions
  • Is there just one Zero?
  • How to set up a local VPN-connected environment with a public IP address?
  • Number of roots of a quadratic form over GF2
more hot questions

Từ khóa » Xn+1-xn =a^n